(New page: Category:ECE Category:QE Category:CNSIP Category:problem solving Category:random variables Category:probability <center> <font size= 4> [[ECE_PhD_Qualifying_Exams...)
 
 
(19 intermediate revisions by 2 users not shown)
Line 23: Line 23:
 
----
 
----
 
=Part 2=
 
=Part 2=
Consider <math class="inline">n</math> independent flips of a coin having probability <math class="inline">p</math> of landing on heads. Say that a changeover occurs whenever an outcome differs from the one preceding it. For instance, if <math class="inline">n=5</math> and the sequence <math class="inline">HHTHT</math> is observed, then there are 3 changeovers. Find the expected number of changeovers for <math class="inline">n</math> flips. ''Hint'': Express the number of changeovers as a sum of Bernoulli random variables.
+
Let <math>X_1,X_2,...</math> be a sequence of jointly Gaussian random variables with covariance
 +
 
 +
<math>Cov(X_i,X_j) = \left\{ \begin{array}{ll}
 +
{\sigma}^2, & i=j\\
 +
\rho{\sigma}^2, & |i-j|=1\\
 +
0, & otherwise
 +
  \end{array} \right.</math>
 +
 
 +
Suppose we take 2 consecutive samples from this sequence to form a vector <math>X</math>, which is then linearly transformed to form a 2-dimensional random vector <math>Y=AX</math>. Find a matrix <math>A</math> so that the components of <math>Y</math> are independent random variables You must justify your answer.
 
----
 
----
 
=Solution 1=
 
=Solution 1=
  
The number of changeovers <math>Y</math> can be expressed as the sum of n-1 Bernoulli random variables:
+
Suppose
  
<math>Y=\sum_{i=1}^{n-1}X_i</math>.
+
<math>A=\left(\begin{array}{cc}
 +
a & b\\
 +
c & d
 +
\end{array} \right)</math>.
  
Therefore,
+
Then the new 2-D random vector can be expressed as
  
<math>E(Y)=E(\sum_{i=1}^{n-1}X_i)=\sum_{i=1}^{n-1}E(X_i)</math>.
+
<math>Y=\left(\begin{array}{c}Y_1 \\ Y_2\end{array} \right)=A\left(\begin{array}{c}X_i \\ X_j\end{array} \right)=\left(\begin{array}{c}aX_i+bX_j \\ cX_i+dX_j\end{array} \right)</math>
  
For Bernoulli random variables,
 
  
<math>E(X_i)=p(E_i=1)=p(1-p)+(1-p)p=2p(1-p)</math>.
+
Therefore,
  
Thus
+
<math>\begin{array}{l}Cov(Y_1,Y_2)=E[(aX_i+bX_j-E(aX_i+bX_j))(cX_i+dX_j-E(cX_i+dX_j))] \\
 +
=E[(aX_i+bX_j-aE(X_i)-bE(X_j))(cX_i+dX_j-cE(X_i)-dE(X_j))] \\
 +
=E[acX_i^2+adX_iX_j-acX_iE(X_i)-adX_iE(X_j)+bcX_iX_j+bdX_j^2-bcX_jE(X_i)\\
 +
-bdX_jE(X_j)-acX_iE(X_i)-adX_jE(X_i)+acE(X_i)^2+adE(X_i)E(X_j)\\
 +
-bcX_iE(X_j)-bdX_jE(X_j)+bcE(X_i)E(X_j)+bdE(X_i)^2]\\
 +
=E(ac(X_i-E(X_i))^2+(ad+bc)(X_i-E(X_i)(X_j-E(X_j))+bd(X_j-E(X_j))^2]\\
 +
=(ac)Cov(X_i,X_i)+(ad+bc)Cov(X-i,X_j)+(bd)Cov(X_j,X_j)\\
 +
=ac\sigma^2+(ad+bc)\rho\sigma^2+bd\sigma^2
 +
\end{array}</math>
  
<math>E(Y)=2(n-1)p(1-p)</math>.
+
Let the above formula equal to 0 and <math>a=b=d=1</math>, we get <math>c=-1</math>.
 +
 
 +
Therefore, a solution is
 +
 
 +
<math>A=\left(\begin{array}{cc}
 +
1 & 1\\
 +
-1 & 1
 +
\end{array} \right)</math>.
 +
 
 +
<font color="red"><u>'''Comments on solution 1'''</u>
 +
 
 +
More procedures and explanations would be better.
 +
 
 +
</font>
  
 
----
 
----
 
==Solution 2==
 
==Solution 2==
  
For <math>n</math> flips, there are <math>n-1</math> changeovers at most. Assume random variable <math>k_i</math> for changeover,
+
Assume  
  
<math>p({k_i}=1)=p(1-p)+(1-p)p=2p(1-p)</math>
+
<math>Y=\left(\begin{array}{c}Y_i \\ Y_j\end{array} \right)=A\left(\begin{array}{c}X_i \\ X_j\end{array} \right)=\left(\begin{array}{c}a_{11}X_i+a_{12}X_j \\ a_{21}X_i+a_{22}X_j\end{array} \right)</math>.
 +
 
 +
Then
 +
 
 +
<math>\begin{array}{l}E(Y_iY_j)=E[(a_{11}X_i+a_{12}X_j)(a_{21}X_i+a_{22}X_j)]\\
 +
=a_{11}a_{21}\sigma^2+a_{12}a_{22}\sigma^2+(a_{11}a_{21}+a_{22}a_{11})E(X_iX_j)
 +
\end{array}</math>
 +
 
 +
For <math>|i-j|\geq1</math>, <math>E(X_i,X_j)=0</math>. Therefore, <math>a_{11}a_{21}+a_{12}a_{22}=0</math>.
 +
 
 +
One solution can be
 +
 
 +
<math>A=\left(\begin{array}{cc}
 +
1 & -1\\
 +
1 & 1
 +
\end{array} \right)</math>.
  
<math>E(k)=\sum_{i_1}^{n-1}p(k_i=1)=2(n-1)p(p-1)</math>
 
  
 
<font color="red"><u>'''Critique on Solution 2:'''</u>  
 
<font color="red"><u>'''Critique on Solution 2:'''</u>  
  
It might be better to claim the changeover as a Bernoulli random variable so the logic is easier to understand.
+
1. <math>E(Y_iY_j)=0</math> is not the condition for the two random variables to be independent.
 +
 
 +
2. "For <math>|i-j|\geq1</math>, <math>E(X_i,X_j)=0</math>" is not supported by the given conditions.
  
 
</font>
 
</font>
 +
----
 +
==Solution 3==
 +
<math>Y=\left(\begin{array}{c}Y_1 \\ Y_2\end{array} \right)=AX=\left(\begin{array}{cc}
 +
a & b\\
 +
c & d
 +
\end{array} \right)\left(\begin{array}{c}X_i \\ X_j\end{array} \right)=\left(\begin{array}{c}aX_i+bX_j \\ cX_i+dX_j\end{array} \right)</math>
 +
 +
We know that the sum of two independent Gaussian distributed random variables is still Gaussian distributed.
 +
 +
Thus, <math>Y_1,Y_2</math> are Gaussian distributed random variables. If they are uncorrelated, then they are also independent.
 +
 +
<math>r = \frac{COV(Y_1,Y_2)}{\sigma1\sigma2} = 0</math>
 +
 +
which indicates that
 +
 +
<math>E(Y_1Y_2) - E(Y_1)E(Y_2) = 0</math>
 +
 +
we know that <math>|i-j|=1</math>
 +
 +
<math>E(Y_1Y_2) = E((aX_i+bX_j)(cX_i+dX_j)) = E(acX_iX_i+adX_iX_j+bcX_jX_i+bdX_jX_j) \\
 +
= ac({\sigma}^{2}+{E(X_i)}^2)+ad(\rho{\sigma}^{2}+E(X_j)E(X_i))+bc(\rho{\sigma}^{2}+E(X_j)E(X_i))+bd({\sigma}^{2}+{E(X_j)}^2) = 0</math>
 +
 +
<math>E(Y_1)E(Y_2) = E((aX_i+bX_j))E((cX_i+dX_j)) = (aE(X_i)+bE(X_j))(cE(X_i)+dE(X_j)) \\
 +
= ac{E(X_i)}^{2}+adE(X_i)E(X_j)+bcE(X_i)E(X_j)+bd{E(X_j)}^{2}</math>
 +
 +
Therefore,
 +
 +
<math>E(Y_1Y_2) - E(Y_1)E(Y_2) = ac{\sigma}^{2}+ad\rho{\sigma}^{2}+bc\rho{\sigma}^{2}+bd{\sigma}^{2} = 0</math>
 +
 +
<math>\left\{ \begin{array}{ll}  ac = -bd\\  ad=-bc  \end{array} \right.</math>
 +
 +
Thus, one of the possible solutions is <math>a = 1,b = -1,c = 1, d = 1</math>
 +
 +
=Similar Question=
 +
Let <math>X_1,X_2,...</math> be a sequence of jointly Gaussian random variables with the same mean <math>u</math> and with covariance function where <math>|\rho|<1</math>
 +
 +
<math>Cov(X_i,X_j) = \left\{ \begin{array}{ll}
 +
{\sigma}^2, & i=j\\
 +
\rho{\sigma}^2, & |i-j|=1\\
 +
0, & otherwise
 +
  \end{array} \right.</math>
 +
 +
Find the mean and variance of <math>S_n = X_1 + ...+ X_n</math>
 
----
 
----
 
[[ECE-QE_CS1-2013|Back to QE CS question 1, August 2013]]
 
[[ECE-QE_CS1-2013|Back to QE CS question 1, August 2013]]
  
 
[[ECE_PhD_Qualifying_Exams|Back to ECE Qualifying Exams (QE) page]]
 
[[ECE_PhD_Qualifying_Exams|Back to ECE Qualifying Exams (QE) page]]

Latest revision as of 16:43, 24 February 2017


ECE Ph.D. Qualifying Exam

Communication, Networking, Signal and Image Processing (CS)

Question 1: Probability and Random Processes

August 2013



Part 2

Let $ X_1,X_2,... $ be a sequence of jointly Gaussian random variables with covariance

$ Cov(X_i,X_j) = \left\{ \begin{array}{ll} {\sigma}^2, & i=j\\ \rho{\sigma}^2, & |i-j|=1\\ 0, & otherwise \end{array} \right. $

Suppose we take 2 consecutive samples from this sequence to form a vector $ X $, which is then linearly transformed to form a 2-dimensional random vector $ Y=AX $. Find a matrix $ A $ so that the components of $ Y $ are independent random variables You must justify your answer.


Solution 1

Suppose

$ A=\left(\begin{array}{cc} a & b\\ c & d \end{array} \right) $.

Then the new 2-D random vector can be expressed as

$ Y=\left(\begin{array}{c}Y_1 \\ Y_2\end{array} \right)=A\left(\begin{array}{c}X_i \\ X_j\end{array} \right)=\left(\begin{array}{c}aX_i+bX_j \\ cX_i+dX_j\end{array} \right) $


Therefore,

$ \begin{array}{l}Cov(Y_1,Y_2)=E[(aX_i+bX_j-E(aX_i+bX_j))(cX_i+dX_j-E(cX_i+dX_j))] \\ =E[(aX_i+bX_j-aE(X_i)-bE(X_j))(cX_i+dX_j-cE(X_i)-dE(X_j))] \\ =E[acX_i^2+adX_iX_j-acX_iE(X_i)-adX_iE(X_j)+bcX_iX_j+bdX_j^2-bcX_jE(X_i)\\ -bdX_jE(X_j)-acX_iE(X_i)-adX_jE(X_i)+acE(X_i)^2+adE(X_i)E(X_j)\\ -bcX_iE(X_j)-bdX_jE(X_j)+bcE(X_i)E(X_j)+bdE(X_i)^2]\\ =E(ac(X_i-E(X_i))^2+(ad+bc)(X_i-E(X_i)(X_j-E(X_j))+bd(X_j-E(X_j))^2]\\ =(ac)Cov(X_i,X_i)+(ad+bc)Cov(X-i,X_j)+(bd)Cov(X_j,X_j)\\ =ac\sigma^2+(ad+bc)\rho\sigma^2+bd\sigma^2 \end{array} $

Let the above formula equal to 0 and $ a=b=d=1 $, we get $ c=-1 $.

Therefore, a solution is

$ A=\left(\begin{array}{cc} 1 & 1\\ -1 & 1 \end{array} \right) $.

Comments on solution 1

More procedures and explanations would be better.


Solution 2

Assume

$ Y=\left(\begin{array}{c}Y_i \\ Y_j\end{array} \right)=A\left(\begin{array}{c}X_i \\ X_j\end{array} \right)=\left(\begin{array}{c}a_{11}X_i+a_{12}X_j \\ a_{21}X_i+a_{22}X_j\end{array} \right) $.

Then

$ \begin{array}{l}E(Y_iY_j)=E[(a_{11}X_i+a_{12}X_j)(a_{21}X_i+a_{22}X_j)]\\ =a_{11}a_{21}\sigma^2+a_{12}a_{22}\sigma^2+(a_{11}a_{21}+a_{22}a_{11})E(X_iX_j) \end{array} $

For $ |i-j|\geq1 $, $ E(X_i,X_j)=0 $. Therefore, $ a_{11}a_{21}+a_{12}a_{22}=0 $.

One solution can be

$ A=\left(\begin{array}{cc} 1 & -1\\ 1 & 1 \end{array} \right) $.


Critique on Solution 2:

1. $ E(Y_iY_j)=0 $ is not the condition for the two random variables to be independent.

2. "For $ |i-j|\geq1 $, $ E(X_i,X_j)=0 $" is not supported by the given conditions.


Solution 3

$ Y=\left(\begin{array}{c}Y_1 \\ Y_2\end{array} \right)=AX=\left(\begin{array}{cc} a & b\\ c & d \end{array} \right)\left(\begin{array}{c}X_i \\ X_j\end{array} \right)=\left(\begin{array}{c}aX_i+bX_j \\ cX_i+dX_j\end{array} \right) $

We know that the sum of two independent Gaussian distributed random variables is still Gaussian distributed.

Thus, $ Y_1,Y_2 $ are Gaussian distributed random variables. If they are uncorrelated, then they are also independent.

$ r = \frac{COV(Y_1,Y_2)}{\sigma1\sigma2} = 0 $

which indicates that

$ E(Y_1Y_2) - E(Y_1)E(Y_2) = 0 $

we know that $ |i-j|=1 $

$ E(Y_1Y_2) = E((aX_i+bX_j)(cX_i+dX_j)) = E(acX_iX_i+adX_iX_j+bcX_jX_i+bdX_jX_j) \\ = ac({\sigma}^{2}+{E(X_i)}^2)+ad(\rho{\sigma}^{2}+E(X_j)E(X_i))+bc(\rho{\sigma}^{2}+E(X_j)E(X_i))+bd({\sigma}^{2}+{E(X_j)}^2) = 0 $

$ E(Y_1)E(Y_2) = E((aX_i+bX_j))E((cX_i+dX_j)) = (aE(X_i)+bE(X_j))(cE(X_i)+dE(X_j)) \\ = ac{E(X_i)}^{2}+adE(X_i)E(X_j)+bcE(X_i)E(X_j)+bd{E(X_j)}^{2} $

Therefore,

$ E(Y_1Y_2) - E(Y_1)E(Y_2) = ac{\sigma}^{2}+ad\rho{\sigma}^{2}+bc\rho{\sigma}^{2}+bd{\sigma}^{2} = 0 $

$ \left\{ \begin{array}{ll} ac = -bd\\ ad=-bc \end{array} \right. $

Thus, one of the possible solutions is $ a = 1,b = -1,c = 1, d = 1 $

Similar Question

Let $ X_1,X_2,... $ be a sequence of jointly Gaussian random variables with the same mean $ u $ and with covariance function where $ |\rho|<1 $

$ Cov(X_i,X_j) = \left\{ \begin{array}{ll} {\sigma}^2, & i=j\\ \rho{\sigma}^2, & |i-j|=1\\ 0, & otherwise \end{array} \right. $

Find the mean and variance of $ S_n = X_1 + ...+ X_n $


Back to QE CS question 1, August 2013

Back to ECE Qualifying Exams (QE) page

Alumni Liaison

Followed her dream after having raised her family.

Ruth Enoch, PhD Mathematics